1975 AHSME Problems/Problem 16

Revision as of 17:16, 19 January 2021 by Hashtagmath (talk | contribs) (Created page with "==Problem== If the first term of an infinite geometric series is a positive integer, the common ratio is the reciprocal of a positive integer, and the sum of the series is <ma...")
(diff) ← Older revision | Latest revision (diff) | Newer revision → (diff)

Problem

If the first term of an infinite geometric series is a positive integer, the common ratio is the reciprocal of a positive integer, and the sum of the series is $3$, then the sum of the first two terms of the series is

$\textbf{(A)}\ \frac{1}{3} \qquad \textbf{(B)}\ \frac{2}{3} \qquad \textbf{(C)}\ \frac{8}{3} \qquad \textbf{(D)}\ 2           \qquad \textbf{(E)}\ \frac{9}{2} \qquad$